Difference between revisions of "2006 AIME A Problems/Problem 4"

(Redirected page to 2006 AIME I Problems/Problem 4)
 
(5 intermediate revisions by 3 users not shown)
Line 1: Line 1:
== Problem ==
+
#REDIRECT [[2006 AIME I Problems/Problem 4]]
Let <math> (a_1,a_2,a_3,\ldots,a_{12}) </math> be a permutation of <math> (1,2,3,\ldots,12) </math> for which
 
 
 
<center><math> a_1>a_2>a_3>a_4>a_5>a_6 \mathrm{\  and \ } a_6<a_7<a_8<a_9<a_{10}<a_{11}<a_{12}. </math></center>
 
 
 
An example of such a permutation is <math> (6,5,4,3,2,1,7,8,9,10,11,12). </math> Find the number of such permutations.
 
 
 
== Solution ==
 
 
 
== See also ==
 
*[[2006 AIME II Problems]]
 
 
 
[[Category:Intermediate Combinatorics Problems]]
 

Latest revision as of 12:03, 28 June 2009